Вы находитесь на странице: 1из 126

GATE CLOUD

NETWORK ANALYSIS
Vol 1

GATE CLOUD

NETWORK ANALYSIS
Vol 1

R. K. Kanodia Ashish Murolia

JHUNJHUNUWALA
JAIPUR

Exclusive Sales Counter & BO: NODIA & COMPANY


55, Suryalok Complex, Gunfoundry, Abids, Hyderabad - 500001. Phone : 040 - 64582577

GATE CLOUD Network Analysis Vol 1, 1e R. K. Kanodia, Ashish Murolia


CC1015 Copyright by Jhunjhunuwala ISBN 9-788192-34834-6

Information contained in this book has been obtained by author, from sources believes to be reliable. However, neither Jhunjhunuwala nor its author guarantee the accuracy or completeness of any information herein, and Jhunjhunuwala nor its author shall be responsible for any error, omissions, or damages arising out of use of this information. This book is published with the understanding that Jhunjhunuwala and its author are supplying information but are not attempting to render engineering or other professional services.

JHUNJHUNUWALA
B-8, Dhanshree Tower Ist, Central Spine, Vidyadhar Nagar, Jaipur 302023 Ph : +91-141-2101150. www.nodia.co.in email : enquiry@nodia.co.in

Printed By: Nodia and Company, Jaipur

Preface to First Edition


GATE CLOUD caters a versatile collection of Multiple Choice Questions to the students who are preparing for GATE (Gratitude Aptitude Test in Engineering) examination. This book contains over 1500 multiple choice solved problems for the subject of Network Analysis, which has a significant weightage in the GATE examination of Electronics and Communication Engineering. The GATE examination is based on multiple choice problems which are tricky, conceptual and tests the basic understanding of the subject. So, the problems included in the book are designed to be as exam-like as possible. The solutions are presented using step by step methodology which enhance your problem solving skills. The book is categorized into fifteen chapters covering all the topics of syllabus of the examination. Each chapter contains : Exercise 1 : Level 1 Exercise 2 : Level 2 Exercise 3 : Mixed Questions Taken form Previous Examinations of GATE. Detailed Solutions to Exercise 1, 2 and 3. Although we have put a vigorous effort in preparing this book, some errors may have crept in. We shall appreciate and greatly acknowledge the comments, criticism and suggestion from the users of this book which leads to some improvement. You may write to us at rajkumar.kanodia@gmail.com and ashish.murolia@gmail.com. Wish you all the success in conquering GATE. Authors

SYLLABUS

GATE ELECTRONICS & COMMUNICATION ENGINEERING


Networks:
Network graphs: matrices associated with graphs; incidence, fundamental cut set and fundamental circuit matrices. Solution methods: nodal and mesh analysis. Network theorems: superposition, Thevenin and Norton's maximum power transfer, Wye-Delta transformation. Steady state sinusoidal analysis using phasors. Linear constant coefficient differential equations; time domain analysis of simple RLC circuits, Solution of network equations using Laplace transform: frequency domain analysis of RLC circuits. 2-port network parameters: driving point and transfer functions. State equations for networks.

IES ELECTRONICS & TELECOMMUNICATION ENGINEERING


Networks Theory:
Network analysis techniques; Network theorems, transient response, steady state sinusoidal response; Network graphs and their applications in network analysis; Tellegens theorem. Two port networks; Z, Y, h and transmission parameters. Combination of two ports, analysis of common two ports. Network functions : parts of network functions, obtaining a network function from a given part. Transmission criteria : delay and rise time, Elmores and other definitions effect of cascading. Elements of network synthesis.

CONTENTS

*******

CHAPTER 5
CIRCUIT THEOREMS

266

Circuit Theorems

Chap 5

EXERCISE 5.1

MCQ 5.1.1

In the network of figure for Vs = V0 , I = 1 A then what is the value of I1 , if Vs = 2V0 ?

(A) 2 A (C) 3 A
MCQ 5.1.2

(B) 1.5 A (D) 2.5 A

In the network of figure, If Is = I 0 then V = 1 volt. What is the value of I1 if Is = 2I 0 ?

(A) 1.5 A (C) 4.5 A


MCQ 5.1.3

(B) 2 A (D) 3 A

The linear network in the figure contains resistors and dependent sources only. When Vs = 10 V , the power supplied by the voltage source is 40 W. What will be the power supplied by the source if Vs = 5 V ?

(A) 20 W (C) 40 W

(B) 10 W (D) can not be determined

Chap 5
MCQ 5.1.4

Circuit Theorems

267

In the circuit below, it is given that when Vs = 20 V , IL = 200 mA . What values of IL and Vs will be required such that power absorbed by RL is 2.5 W ?

(A) 1 A , 2.5 V (C) 0.5 A, 50 V


MCQ 5.1.5

(B) 0.5 A, 2 V (D) 2 A, 1.25 V

For the circuit shown in figure below, some measurements are made and listed in the table.

Which of the following equation is true for IL ? (A) IL = 0.6Vs + 0.4Is (B) IL = 0.2Vs 0.3Is (C) IL = 0.2Vs + 0.3Is (D) IL = 0.4Vs 0.6Is
MCQ 5.1.6

In the circuit below, the voltage drop across the resistance R2 will be equal to

(A) 46 volt (C) 22 volt

(B) 38 volt (D) 14 volt

268
MCQ 5.1.7

Circuit Theorems

Chap 5

In the circuit below, the voltage V across the 40 resistor would be equal to

(A) 80 volt (C) 160 volt


MCQ 5.1.8

(B) 40 volt (D) zero

In the circuit below, current I = I1 + I2 + I 3 , where I1 , I2 and I 3 are currents due to 60 A, 30 A and 30 V sources acting alone. The values of I1 , I2 and I 3 are respectively

(A) 8 A, 8 A, 4 A (B) 12 A, 12 A, 5 A (C) 4 A, 4 A, 1 A (D) 2 A, 2 A, 4 A


MCQ 5.1.9

The value of current I flowing through 2 resistance in the circuit below, equals to

(A) 10 A (C) 4 A
MCQ 5.1.10

(B) 5 A (D) zero

In the circuit below, current I is equal to sum of two currents I1 and I2 . What are the values of I1 and I2 ?

Chap 5

Circuit Theorems

269

(A) 6 A, 1 A (C) 3 A, 1 A
MCQ 5.1.11

(B) 9 A, 6 A (D) 3 A, 4 A

A network consists only of independent current sources and resistors. If the values of all the current sources are doubled, then values of node voltages (A) remains same (B) will be doubled (C) will be halved (D) changes in some other way.

MCQ 5.1.12

Consider a network which consists of resistors and voltage sources only. If the values of all the voltage sources are doubled, then the values of mesh current will be (A) doubled (B) same (C) halved (D) none of these

MCQ 5.1.13

In the circuit shown in the figure below, the value of current I will be be given by

(A) 1.5 A (C) 0.05 A


MCQ 5.1.14

(B) 0.3 A (D) 0.5 A

What is the value of current I in the following network ?

270

Circuit Theorems

Chap 5

(A) 4 A (C) 2 A
MCQ 5.1.15

(B) 6 A (D) 1 A

In the given network if V1 = V2 = 0 , then what is the value of Vo ?

(A) 3.2 V (B) 8 V (C) 5.33 V (D) zero


MCQ 5.1.16

The value of current I in the circuit below is equal to

(A)

2 7

(B) 1 A (C) 2 A (D) 4 A


MCQ 5.1.17

What is the value of current I in the circuit shown below ?

(A) 8.5 A (C) 1.5 A

(B) 4.5 A (D) 5.5 A

Chap 5
MCQ 5.1.18

Circuit Theorems

271

In the circuit below, the 12 V source

(A) absorbs 36 W (C) absorbs 100 W


MCQ 5.1.19

(B) delivers 4 W (D) delivers 36 W

Which of the following circuits is equivalent to the circuit shown below ?

MCQ 5.1.20

Consider a dependent current source shown in figure below.

272

Circuit Theorems

Chap 5

The source transformation of above is given by

MCQ 5.1.21

Consider a circuit shown in the figure

Which of the following circuit is equivalent to the above circuit ?

Chap 5
MCQ 5.1.22

Circuit Theorems

273

How much power is being dissipated by the 4 k resistor in the network ?

(A) 0 W (C) 9 mW
MCQ 5.1.23

(B) 2.25 mW (D) 4 mW

For the circuit shown in the figure the Thevenin voltage and resistance seen from the terminal a -b are respectively

(A) 34 V, 0 (B) 20 V , 24 (C) 14 V, 0 (D) 14 V , 24


MCQ 5.1.24

The Thevenin equivalent resistance RTh between the nodes a and b in the following circuit is

(A) 3 (B) 16 (C) 12 (D) 4

274
MCQ 5.1.25

Circuit Theorems

Chap 5

In the following circuit, Thevenin voltage and resistance across terminal a and b respectively are

(A) 10 V, 18 (C) 10 V, 18.67


MCQ 5.1.26

(B) 2 V, 18 (D) 2 V, 18.67

The value of RTh and VTh such that the circuit of figure (B) is the Thevenin equivalent circuit of the circuit shown in figure (A), will be equal to

(A) RTh = 6 , VTh = 4 V (C) RTh = 2 , VTh = 24 V


MCQ 5.1.27

(B) RTh = 6 , VTh = 28 V (D) RTh = 10 , VTh = 14 V

What values of RTh and VTh will cause the circuit of figure (B) to be the equivalent circuit of figure (A) ?

(A) 2.4 , 24 V (C) 10 , 24 V

(B) 3 , 16 V (D) 10 , 24 V

Chap 5

Circuit Theorems

275

Common Data for Q. 28 to 29 :


Consider the two circuits shown in figure (A) and figure (B) below

MCQ 5.1.28

The value of Thevenin voltage across terminals a -b of figure (A) and figure (B) respectively are (A) 30 V, 36 V (B) 28 V, 12 V (C) 18 V, 12 V (D) 30 V, 12 V

MCQ 5.1.29

The value of Thevenin resistance across terminals a -b of figure (A) and figure (B) respectively are (A) zero, 3 (B) 9 , 16 (C) 2 , 3 (D) zero, 16

Statement for linked Questions 30 and 31 :


Consider the circuit shown in the figure.

MCQ 5.1.30

The equivalent Thevenin voltage across terminal a -b is (A) 31.2 V (B) 19.2 V (C) 16.8 V (D) 24 V

MCQ 5.1.31

The Norton equivalent current with respect to terminal a -b is (A) 13 A (B) 7 A (C) 8 A (D) 10 A

276
MCQ 5.1.32

Circuit Theorems

Chap 5

For a network having resistors and independent sources, it is desired to obtain Thevenin equivalent across the load which is in parallel with an ideal current source. Then which of the following statement is true ? (A) The Thevenin equivalent circuit is simply that of a voltage source. (B) The Thevenin equivalent circuit consists of a voltage source and a series resistor. (C) The Thevenin equivalent circuit does not exist but the Norton equivalent does exist. (D) None of these

MCQ 5.1.33

The Thevenin equivalent circuit of a network consists only of a resistor (Thevenin voltage is zero). Then which of the following elements might be contained in the network ? (A) resistor and independent sources (B) resistor only (C) resistor and dependent sources (D) resistor, independent sources and dependent sources.

MCQ 5.1.34

In the following network, value of current I through 6 resistor is given by

(A) 0.83 A (C) 1 A


MCQ 5.1.35

(B) 2 A (D) 0.5 A

For the circuit shown in the figure, the Thevenins voltage and resistance looking into a -b are

(A) 2 V, 3 (C) 6 V, 9

(B) 2 V, 2 (D) 6 V, 3

Chap 5
MCQ 5.1.36

Circuit Theorems

277

For the circuit below, what value of R will cause I = 3 A ?

(A) 2/3 (C) zero


MCQ 5.1.37

(B) 4 (D) none of these

For the following circuit, values of voltage V for different values of R are given in the table.

The Thevenin voltage and resistance of the unknown circuit are respectively. (A) 14 V, 4 (B) 4 V, 1 (C) 14 V, 6 (D) 10 V, 2
MCQ 5.1.38

In the circuit shown below, the Norton equivalent current and resistance with respect to terminal a -b is

(A)

17 6

A, 0

(B) 2 A, 24 (C) 7 A , 24 6 (D) 2 A , 24

278
MCQ 5.1.39

Circuit Theorems

Chap 5

The Norton equivalent circuit for the circuit shown in figure is given by

MCQ 5.1.40

What are the values of equivalent Norton current source (IN ) and equivalent resistance (RN ) across the load terminal of the circuit shown in figure ?

(A) (B) (C) (D)


MCQ 5.1.41

IN 10 A 10 A 3.33 A 6.66 A

RN 2 9 9 2

For a network consisting of resistors and independent sources only, it is desired to obtain Thevenins or Nortons equivalent across a load which is in parallel with an ideal voltage sources. Consider the following statements :

Chap 5

Circuit Theorems

279

1. 2. 3.

Thevenin equivalent circuit across this terminal does not exist. The Thevenin equivalent circuit exists and it is simply that of a voltage source. The Norton equivalent circuit for this terminal does not exist.

Which of the above statements is/are true ? (A) 1 and 3 (B) 1 only (C) 2 and 3
MCQ 5.1.42

(D) 3 only

For a network consisting of resistors and independent sources only, it is desired to obtain Thevenins or Nortons equivalent across a load which is in series with an ideal current sources. Consider the following statements 1. Norton equivalent across this terminal is not feasible. 2. 3. Norton equivalent circuit exists and it is simply that of a current source only. Thevenins equivalent circuit across this terminal is not feasible.

Which of the above statements is/are correct ? (A) 1 and 3 (B) 2 and 3 (C) 1 only (D) 3 only
MCQ 5.1.43

The Norton equivalent circuit of the given network with respect to the terminal a -b, is

280
MCQ 5.1.44

Circuit Theorems

Chap 5

The maximum power that can be transferred to the resistance R in the circuit is

(A) 486 mW (B) 243 mW (C) 121.5 mW (D) 225 mW


MCQ 5.1.45

In the circuit below, if RL is fixed and Rs is variable then for what value of Rs power dissipated in RL will be maximum ?

(A) RS = RL (B) RS = 0 (C) RS = RL /2 (D) RS = 2RL


MCQ 5.1.46

In the circuit shown below the maximum power transferred to RL is Pmax , then

(A) RL = 12 , Pmax = 12 W (B) RL = 3 , Pmax = 96 W (C) RL = 3 , Pmax = 48 W (D) RL = 12 , Pmax = 24 W


MCQ 5.1.47

In the circuit shown in figure (A) if current I1 = 2 A , then current I2 and I 3 in figure (B) and figure (C) respectively are

Chap 5

Circuit Theorems

281

(A) 2 A, 2 A (C) 2 A, 2 A
MCQ 5.1.48

(B) 2 A , 2 A (D) 2 A , 2 A

In the circuit of figure (A), if I1 = 20 mA , then what is the value of current I2 in the circuit of figure (B) ?

(A) 40 mA (C) 20 mA
MCQ 5.1.49

(B) 20 mA (D) R1 , R2 and R 3 must be known

If V1 = 2 V in the circuit of figure (A), then what is the value of V2 in the circuit of figure (B) ?

282

Circuit Theorems

Chap 5

(A) 2 V (C) 4 V
MCQ 5.1.50

(B) 2 V (D) R1 , R2 and R 3 must be known

The value of current I in the circuit below is equal to

(A) 100 mA (C) 233.34 mA


MCQ 5.1.51

(B) 10 mA (D) none of these

The value of current I in the following circuit is equal to

(A) 1 A (C) 3 A

(B) 6 A (D) 2 A

***********

Chap 5

Circuit Theorems

283

EXERCISE 5.2

MCQ 5.2.1

A simple equivalent circuit of the two-terminal network shown in figure is

MCQ 5.2.2

For the following circuit the value of RTh is

(A) 3 (C) 6

(B) 12 (D) 3

284
MCQ 5.2.3

Circuit Theorems

Chap 5

If V = AV1 + BV2 + CI 3 in the following circuit, then values of A, B and C respectively are

(A) 2 , 2 , 1 3 3 3 (C) 1 , 1 , 1 2 2 3
MCQ 5.2.4

(B) 1 , 1 , 100 3 3 3 (D) 1 , 2 , 100 3 3 3

What is the value of current I in the network of figure ?

(A) 0.67 A (C) 1.34 A


MCQ 5.2.5

(B) 2 A (D) 0.5 A

The value of current I in the figure is

(A) 1 mA (C) 1.8 mA


MCQ 5.2.6

(B) 1.4 mA (D) 1.2 mA

For the circuit of figure, some measurements were made at the terminals a -b and given in the table below.

Chap 5

Circuit Theorems

285

What is the value of IL for RL = 20 ? (A) 3 A (C) 2 A


MCQ 5.2.7

(B) 12 A (D) 4 A

In the circuit below, for what value of k , load RL = 2 absorbs maximum power ?

(A) 4 (C) 2
MCQ 5.2.8

(B) 7 (D) 6

In the circuit shown below, the maximum power that can be delivered to the load RL is equal to

(A) 72 mW (C) 24 mW
MCQ 5.2.9

(B) 36 mW (D) 18 mW

For the linear network shown below, V -I characteristic is also given in the figure. The value of Norton equivalent current and resistance respectively are

(A) 3 A, 2 (C) 6 A, 0.5

(B) 6 , 2 (D) 3 A, 0.5

286
MCQ 5.2.10

Circuit Theorems

Chap 5

In the following circuit a network and its Thevenin and Norton equivalent are given.

The value of the parameter are VTh (A) (B) (C) (D)
MCQ 5.2.11

RTh 2 2 1.2 5

IN 2A 2A
30 3 8 5

RN 2 3 1.2 5

4V 4V 8V 8V

A A

In the following circuit the value of voltage V1 is

(A) 6 V (C) 8 V
MCQ 5.2.12

(B) 7 V (D) 10 V

A practical DC current source provide 20 kW to a 50 load and 20 kW to a 200 load. The maximum power, that can drawn from it, is (A) 22.5 kW (B) 45 kW (C) 30.3 kW (D) 40 kW

MCQ 5.2.13

For the following circuit the value of equivalent Norton current IN and resistance RN are

(A) 2 A, 20 (C) 0 A, 20

(B) 2 A, 20 (D) 0 A, 20

Chap 5
MCQ 5.2.14

Circuit Theorems

287

Consider the following circuits shown below

The relation between Ia and Ib is (A) Ib = Ia + 6 (C) Ib = 1.5Ia


MCQ 5.2.15

(B) Ib = Ia + 2 (D) Ib = Ia

If I = 5 A in the circuit below, then what is the value of voltage source Vs ?

(A) 28 V (C) 200 V


MCQ 5.2.16

(B) 56 V (D) 224 V

For the following circuit, value of current I is given by

(A) 0.5 A (C) 1 A

(B) 3.5 A (D) 2 A

288

Circuit Theorems

Chap 5

Statement for Linked Questions


In the following circuit, some measurements were made at the terminals a , b and given in the table below.

MCQ 5.2.17

The Thevenin equivalent of the unknown network across terminal a -b is (A) 3 , 14 V (B) 5 , 16 V (C) 16 , 38 V (D) 10 , 26 V

MCQ 5.2.18

The value of R that will cause I to be 1 A, is (A) 22 (B) 16 (C) 8 (D) 11 In the circuit shown in fig (a) if current I1 = 2.5 A then current I2 and I 3 in fig (B) and (C) respectively are

MCQ 5.2.19

(A) 5 A, 10 A (C) 5 A, 10 A

(B) 5 A , 10 A (D) 5 A , 10 A

Chap 5
MCQ 5.2.20

Circuit Theorems

289

The Thevenin equivalent resistance between terminal a and b in the following circuit is

(A) 22 (B) 11 (C) 17 (D) 1


MCQ 5.2.21

In the circuit shown below, the value of current I will be given by

(A) 2.5 A (B) 1.5 A (C) 4 A (D) 2 A


MCQ 5.2.22

The V -I relation of the unknown element X in the given network is V = AI + B . The value of A (in ohm) and B (in volt) respectively are

(A) 2, 20 (C) 0.5, 4

(B) 2, 8 (D) 0.5, 16

290
MCQ 5.2.23

Circuit Theorems

Chap 5

The power delivered by 12 V source in the following network is

(A) 24 W (C) 120 W


MCQ 5.2.24

(B) 96 W (D) 48 W

For the following network the V -I curve with respect to terminals a -b, is given by

Chap 5
MCQ 5.2.25

Circuit Theorems

291

In the circuit shown, what value of RL maximizes the power delivered to RL ?

(A) 286 (C) zero


MCQ 5.2.26

(B) 350 (D) 500

The V -I relation for the circuit below is plotted in the figure. The maximum power that can be transferred to the load RL will be

(A) 4 mW (C) 2 mW
MCQ 5.2.27

(B) 8 mW (D) 16 mW

In the following circuit equivalent Thevenin resistance between nodes a and b is RTh = 3 . The value of is

(A) 2 (C) 3
MCQ 5.2.28

(B) 1 (D) 4

A network N feeds a resistance R as shown in circuit below. Let the power consumed by R be P . If an identical network is added as shown in figure, the power consumed by R will be

292

Circuit Theorems

Chap 5

(A) equal to P (C) between P and 4P


MCQ 5.2.29

(B) less than P (D) more than 4P

A certain network consists of a large number of ideal linear resistors, one of which is R and two constant ideal source. The power consumed by R is P1 when only the first source is active, and P2 when only the second source is active. If both sources are active simultaneously, then the power consumed by R is (B) P1 ! P2 (A) P1 ! P2 (C) ( P1 ! P2 ) 2 (D) (P1 ! P2) 2

MCQ 5.2.30

If the 60 resistance in the circuit of figure (A) is to be replaced with a current source Is and 240 shunt resistor as shown in figure (B), then magnitude and direction of required current source would be

(A) 200 mA, upward (C) 50 mA, downward


MCQ 5.2.31

(B) 150 mA, downward (D) 150 mA, upward

The Thevenins equivalent of the circuit shown in the figure is

(A) 4 V, 48 (C) 24 V, 24

(B) 24 V, 12 (D) 12 V, 12

Chap 5
MCQ 5.2.32

Circuit Theorems

293

The voltage VL across the load resistance in the figure is given by VL = V b RL l R + RL V and R will be equal to

(A) 10 V , 2 (C) 10 V , 2
MCQ 5.2.33

(B) 10 V, 2 (D) none of these

The maximum power that can be transferred to the load resistor RL from the current source in the figure is

(A) 4 W (C) 16 W

(B) 8 W (D) 2 W

Common data for Q. 34 to Q. 35


An electric circuit is fed by two independent sources as shown in figure.

MCQ 5.2.34

The power supplied by 36 V source will be (A) 108 W (B) 162 W (C) 129.6 W (D) 216 W

294
MCQ 5.2.35

Circuit Theorems

Chap 5

The power supplied by 27 A source will be (A) 972 W (B) 1083 W (C) 1458 W (D) 1026 W

MCQ 5.2.36

In the circuit shown in the figure, power dissipated in 4 resistor is

(A) 225 W (C) 9 W


MCQ 5.2.37

(B) 121 W (D) none of these

In the circuit given below, viewed from a -b, the circuit can be reduced to an equivalent circuit as

(A) 10 volt source in series with 2 k resistor (B) 1250 resistor only (C) 20 V source in series with 1333.34 resistor (D) 800 resistor only
MCQ 5.2.38

What is the value of voltage V in the following network ?

Chap 5

Circuit Theorems

295

(A) 14 V (C) 10 V
MCQ 5.2.39

(B) 28 V (D) none of these

For the circuit shown in figure below the value of RTh is

(A) 100 (C) 200


MCQ 5.2.40

(B) 136.4 (D) 272.8

Consider the network shown below :

The power absorbed by load resistance RL is shown in table : RL P 10 k 3.6 mW 30 k 4.8 mW

The value of RL , that would absorb maximum power, is (A) 60 k (B) 100 (C) 300
MCQ 5.2.41

(D) 30 k

The V -I equation for the network shown in figure, is given by

296

Circuit Theorems

Chap 5

(A) 7V = 200I + 54 (C) V = 200I + 54


MCQ 5.2.42

(B) V = 100I + 36 (D) V = 50I + 54

In the following circuit the value of open circuit voltage and Thevenin resistance at terminals a, b are

(A) Voc = 100 V , RTh = 1800 (B) Voc = 0 V , RTh = 270 (C) Voc = 100 V , RTh = 90 (D) Voc = 0 V , RTh = 90

***********

Chap 5

Circuit Theorems

297

EXERCISE 5.3

Common Data for Questions 1 and 2 :


GATE EC 2012

With 10 V dc connected at port A in the linear nonreciprocal two-port network shown below, the following were observed : (i) 1 connected at port B draws a current of 3 A (ii) 2.5 connected at port B draws a current of 2 A

GATE EE 2012

MCQ 5.3.1

With 10 V dc connected at port A, the current drawn by 7 connected at port B is (A) 3/7 A (B) 5/7 A (C) 1 A (D) 9/7 A

MCQ 5.3.2

For the same network, with 6 V dc connected at port A, 1 connected at port B draws 7/3 A. If 8 V dc is connected to port A, the open circuit voltage at port B is (A) 6 V (B) 7 V (C) 8 V (D) 9 V

MCQ 5.3.3
GATE EC 2011

In the circuit shown below, the value of RL such that the power transferred to RL is maximum is

(A) 5 (C) 15

(B) 10 (D) 20

298
MCQ 5.3.4
GATE EC 2009

Circuit Theorems

Chap 5

In the circuit shown, what value of RL maximizes the power delivered to RL ?

(A) 2.4 (C) 4


MCQ 5.3.5
GATE EC 2007

(B) 8 3 (D) 6

For the circuit shown in the figure, the Thevenin voltage and resistance looking into X -Y are

(A) (C)
MCQ 5.3.6
GATE EC 2005

4 3 4 3

V, 2 V, 2 3

(B) 4 V, 2 3 (D) 4 V, 2

The maximum power that can be transferred to the load resistor RL from the voltage source in the figure is

(A) 1 W (C) 0.25 W


MCQ 5.3.7
GATE EC 2005

(B) 10 W (D) 0.5 W

For the circuit shown in the figure, Thevenins voltage and Thevenins equivalent resistance at terminals a -b is

Chap 5

Circuit Theorems

299

(A) 5 V and 2 (C) 4 V and 2


MCQ 5.3.8
GATE EC 2002

(B) 7.5 V and 2.5 (D) 3 V and 2.5

In the network of the figure, the maximum power is delivered to RL if its value is

(A) 16 (C) 60
MCQ 5.3.9
GATE EC 2000

(B) 40 3 (D) 20

Use the data of the figure (a). The current i in the circuit of the figure (b)

(A) 2 A (C) 4 A
MCQ 5.3.10
GATE EC 1999

(B) 2 A (D) 4 A

The value of R (in ohms) required for maximum power transfer in the network shown in the given figure is

300

Circuit Theorems

Chap 5

(A) 2 (C) 8
MCQ 5.3.11
GATE EC 1998

(B) 4 (D) 16

Superposition theorem is NOT applicable to networks containing (A) nonlinear elements (B) dependent voltage sources (C) dependent current sources (D) transformers

MCQ 5.3.12
GATE EC 1997

The voltage V in the figure is always equal to

(A) 9 V (C) 1 V
MCQ 5.3.13
GATE EE 1997

(B) 5 V (D) None of the above

The Thevenin voltage and resistance about AB for the circuit shown in figure respectively are

(A) 10 V, 2 9 (C) 10 V,
MCQ 5.3.14
GATE EE 1997
12 5

(B) 0 V, 2 9 (D) 0 V,
12 5

For the circuit shown in figure, the Norton equivalent source current value and and its resistance is

(A) ^2 A, 3 h 2 (C) ^4 A, 3 h 2

(B) ^2 A, 9 h 2 (D) ^4 A, 3 h 4

Chap 5
MCQ 5.3.15
GATE EE 1998

Circuit Theorems

301

Viewed from the terminals A-B , the following circuit shown in figure can be reduced to an equivalent circuit of a single voltage source in series with a single resistor with the following parameters

(A) 5 volt source in series with 10 resistor (B) 1 volt source in series with 2.4 resistor (C) 15 volt source in series with 2.4 resistor (D) 1 volt source in series with 10 resistor

Statement for Linked Answer Question 16 and 17 :

MCQ 5.3.16
GATE EE 2009

For the circuit given above, the Thevenins resistance across the terminals A and B is (A) 0.5 k (B) 0.2 k (C) 1 k (D) 0.11 k

MCQ 5.3.17
GATE EE 2009

For the circuit given above, the Thevenins voltage across the terminals A and B is (A) 1.25 V (B) 0.25 V (C) 1 V (D) 0.5 V

MCQ 5.3.18
GATE EE 2010

As shown in the figure, a 1 resistance is connected across a source that has a load line V + I = 100 . The current through the resistance is

302

Circuit Theorems

Chap 5

(A) 25 A (C) 100 A


MCQ 5.3.19
GATE EE 2011

(B) 50 A (C) 200 A

In the circuit given below, the value of R required for the transfer of maximum power to the load having a resistance of 3 is

(A) zero (C) 6


MCQ 5.3.20
GATE IN 2000

(B) 3 (D) infinity

For the circuit shown in figure VR = 20 V when R = 10 and VR = 30 V when R = 20 . For R = 80 ,VR will read as

(A) 48 V (C) 120 V


MCQ 5.3.21
GATE IN 2000

(B) 60 V (D) 160 V

For the circuit shown in figure R is adjusted to have maximum power transferred to it. The maximum power transferred is

(A) 16 W (C) 64 W
MCQ 5.3.22
GATE IN 2001

(B) 32 W (D) 100 W

In the circuit shown in figure, current through the 5 resistor is

Chap 5

Circuit Theorems

303

(A) zero (C) 3 A


MCQ 5.3.23
GATE IN 2007

(B) 2 A (D) 7 A

In full sunlight, a solar cell has a short circuit current of 75 mA and a current of 70 mA for a terminal voltage of 0.6 with a given load. The Thevenin resistance of the solar cell is (A) 8 (B) 8.6 (C) 120 (D) 240

MCQ 5.3.24
GATE IN 2009

The source network S is connected to the load network L as shown by dashed lines. The power transferred from S to L would be maximum when RL is

(A) 0 (C) 0.8


MCQ 5.3.25
GATE IN 2011

(B) 0.6 (D) 2

The current I shown in the circuit given below is equal to

(A) 3 A (C) 6 A ***********

(B) 3.67 A (D) 9 A

304

Circuit Theorems

Chap 5

SOLUTIONS 5.1

SOL 5.1.1

Option (C) is correct. We solve this problem using principal of linearity.

In the left, 4 and 2 are in series and has same current I = 1 A . V3 = 4I + 2I = 6I = 6 V I 3 = V3 = 6 = 2 A 3 3 I2 = I3 + I = 2+1 = 3A V1 = (1) I2 + V3 = 3+6 = 9V I1 = V1 = 9 = 3 A 6 6 2 Applying principal of linearity For Vs = V0 , I1 = 3 A 2 So for Vs = 2V0 , I1 = 3 # 2 = 3 A 2 Option (D) is correct. We solve this problem using principal of linearity.

(using KVL)

(using ohms law) (using KCL) (using KVL)

(using ohms law)

SOL 5.1.2

Chap 5

Circuit Theorems

305

I =V = 1 = 1A 1 1 V2 = 2I + (1) I =3V I2 = V2 = 3 = 1 A 6 6 2 I1 = I 2 + I = 1+1 = 3 A 2 2 Applying principal of superposition When Is = I 0 , and V = 1 V , So, if Is = 2I 0 ,


SOL 5.1.3

(using ohms law) (using KVL)

(using ohms law) (using KCL)

I1 = 3 A 2 I1 = 3 # 2 = 3 A 2

Option (B) is correct.

Vs = 10 V , P = 40 W So, Is = P = 40 = 4 A Vs 10 Now, Vsl = 5 V , so Isl= 2 A New value of the power supplied by source is Psl = VslIsl= 5 # 2 = 10 W Note: Linearity does not apply to power calculations. For,
SOL 5.1.4

(From linearity)

Option (C) is correct. From linearity, we know that in the circuit Vs ratio remains constant IL Vs = 20 = 100 IL 200 # 103 Let current through load is ILl when the power absorbed is 2.5 W, so PL = (ILl) 2 RL 2.5 = (ILl) 2 # 10 ILl = 0.5 A Vs = Vsl = 100 IL ILl So, Vsl = 100ILl= 100 # 0.5 = 50 V Thus required values are ILl = 0.5 A , Vsl= 50 V

306
SOL 5.1.5

Circuit Theorems

Chap 5

Option (D ) is correct. From linearity, IL = AVs + BIs , From the table 2 = 14A + 6B 6 = 18A + 2B Solving equation (i) & (ii) A = 0.4 , B = 0.6 So, IL = 0.4Vs 0.6Is

A and B are constants ...(i) ...(ii)

SOL 5.1.6

Option (B) is correct. The circuit has 3 independent sources, so we apply superposition theorem to obtain the voltage drop. Due to 16 V source only : (Open circuit 5 A source and Short circuit 32 V source) Let voltage across R2 due to 16 V source only is V1 .

Using voltage division 8 (16) 24 + 8 = 4 V Due to 5 A source only : (Short circuit both the 16 V and 32 V sources) Let voltage across R2 due to 5 A source only is V2 . V1 =

V2 = (24 || 16 || 16 ) # 5 = 6 # 5 = 30 volt Due to 32 V source only : (Short circuit 16 V source and open circuit 5 A source) Let voltage across R2 due to 32 V source only is V3

Chap 5

Circuit Theorems

307

Using voltage division 9.6 (32) = 12 V 16 + 9.6 By superposition, the net voltage across R2 is V = V1 + V2 + V3 = 4 + 30 + 12 = 38 volt Alternate Method: The problem may be solved by applying a node equation at the top node. V3 =
SOL 5.1.7

Option (C) is correct. We solve this problem using superposition. Due to 9 A source only : (Open circuit 6 A source)

Using current division V1 = 20 (9) & V1 = 80 volt 40 20 + (40 + 30) Due to 6 A source only : (Open circuit 9 A source)

Using current division, V2 = 30 (6) & V2 = 80 volt 40 30 + (40 + 20) From superposition, V = V1 + V2 = 80 + 80 = 160 volt Alternate Method: The problem may be solved by transforming both the current sources into equivalent voltage sources and then applying voltage division.
SOL 5.1.8

Option (C) is correct

CONCEPT: Superposition

308

Circuit Theorems

Chap 5

Due to 60 A source only : (Open circuit 30 A and short circuit 30 V sources)

12 || 6 = 4

Using current division 2 (60) = 12 A 2+8 Again, Ia will be distributed between parallel combination of 12 and 6 I1 = 6 (12) = 4 A 12 + 6 Due to 30 A source only : (Open circuit 60 A and short circuit 30 V sources) Ia =

Using current division 4 (30) = 12 A 4+6 Ib will be distributed between parallel combination of 12 and 6 I2 = 6 (12) = 4 A 12 + 6 Due to 30 V source only : (Open circuit 60 A and 30 A sources) Ib =

Chap 5

Circuit Theorems

309

Using source transformation

Using current division I 3 =


SOL 5.1.9

3 (5) = 1 A 12 + 3

Option (B) is correct. Using super position, we obtain I . Due to 10 V source only : (Open circuit 5 A source)

I1 = 10 = 5 A 2 Due to 5 A source only : (Short circuit 10 V source)

I2 = 0 I = I1 + I 2 = 5 + 0 = 5 A

Alternatively :
We can see that voltage source is in parallel with resistor and current source so voltage across parallel branches will be 10 V and I = 10/2 = 5 A

310
SOL 5.1.10

Circuit Theorems

Chap 5

Option (C) is correct. Using superposition, I = I1 + I 2 Let I1 is the current due to 9 A source only. (i.e. short 18 V source)

6 (9) = 3 A (current division) 6 + 12 Let I2 is the current due to 18 V source only (i.e. open 9 A source) I1 =

So,
SOL 5.1.11

18 = 1 A 6 + 12 I1 = 3 A , I 2 = 1 A I2 =

Option (B) is correct. From superposition theorem, it is known that if all source values are doubled, then node voltages also be doubled. Option (A) is correct. From the principal of superposition, doubling the values of voltage source doubles the mesh currents. Option (D) is correct. Applying superposition, Due to 6 V source only : (Open circuit 2 A current source)

SOL 5.1.12

SOL 5.1.13

Chap 5

Circuit Theorems

311

6 = 0.5 A 6+6 Due to 2 A source only : (Short circuit 6 V source) I1 =

6 ( 2) (using current division) 6+6 = 1 A I = I1 + I2 = 0.5 1 = 0.5 A Alternate Method: This problem may be solved by using a single KVL equation around the outer loop. I2 =
SOL 5.1.14

Option (A) is correct. Applying superposition, Due to 24 V source only : (Open circuit 2 A and short circuit 20 V source)

I1 = 24 = 3 A 8

312

Circuit Theorems

Chap 5

Due to 20 V source only : (Short circuit 24 V and open circuit 2 A source)

So (Due to short circuit) I2 = 0 Due to 2 A source only : (Short circuit 24 V and 20 V sources)

4 (2) (using current division) 4+4 = 1A So I = I1 + I 2 + I 3 = 3 + 0 + 1 = 4 A Alternate Method: We can see that current in the middle 4 resistor is I 2 , therefore I can be obtained by applying KVL in the bottom left mesh. I3 =
SOL 5.1.15

Option (D) is correct. V1 = V2 = 0

(short circuit both sources)

Vo = 0
SOL 5.1.16

Option (C) is correct. Using source transformation, we can obtain I in following steps.

Chap 5

Circuit Theorems

313

I = 6 + 8 = 14 = 2 A 3+4 7 Alternate Method: Try to solve the problem by obtaining Thevenin equivalent for right half of the circuit.
SOL 5.1.17

Option (C) is correct. Using source transformation of 48 V source and the 24 V source

using parallel resistances combination

Source transformation of 8 A and 6 A sources

314

Circuit Theorems

Chap 5

SOL 5.1.18

Writing KVL around anticlock wise direction 12 2I + 40 4I 2I 16 = 0 12 8I = 0 I = 12 = 1.5 A 8 Option (D) is correct. Using source transformation of 4 A and 6 V source.

Adding parallel current sources

Source transformation of 5 A source

Applying KVL around the anticlock wise direction 5 I + 8 2I 12 = 0 9 3I = 0 I = 3 A Power absorbed by 12 V source P12 V = 12 # I = 12 # 3 = 36 W or, 12 V source supplies 36 W power.

(Passive sign convention)

Chap 5
SOL 5.1.19

Circuit Theorems

315

Option (B) is correct. We know that source transformation also exists for dependent source, so

Current source values Is = 6Ix = 3Ix (downward) 2 Rs = 2


SOL 5.1.20

Option (C) is correct. We know that source transformation is applicable to dependent source also. Values of equivalent voltage source Vs = (4Ix ) (5) = 20Ix Rs = 5

SOL 5.1.21

Option (C) is correct. Combining the parallel resistance and adding the parallel connected current sources. 9 A 3 A = 6 A (upward) 3 || 6 = 2

Source transformation of 6 A source

316

Circuit Theorems

Chap 5

SOL 5.1.22

Option (B) is correct. We apply source transformation as follows. Transforming 3 mA source into equivalent voltage source and 18 V source into equivalent current source.

6 k and 3 k resistors are in parallel and equivalent to 2 .

Again transforming 3 mA source

I = P4 k

6+6 = 3 mA 2+8+4+2 4 = I 2 (4 # 103)

Chap 5

Circuit Theorems

317

SOL 5.1.23

= b 3 l # 4 = 2.25 mW 4 Option (D) is correct. Thevenin voltage : (Open circuit voltage) The open circuit voltage between a -b can be obtained as
2

Writing KCL at node a VTh 10 + 1 = 0 24 VTh 10 + 24 = 0 VTh = 14 volt Thevenin Resistance : To obtain Thevenins resistance, we set all independent sources to zero i.e., short circuit all the voltage sources and open circuit all the current sources.

RTh = 24
SOL 5.1.24

Option (A) is correct. Set all independent sources to zero (i.e. open circuit current sources and short circuit voltage sources) to obtain RTh

RTh = 12 || 4 = 3
SOL 5.1.25

Option (B) is correct. Thevenin voltage :

318

Circuit Theorems

Chap 5

Using voltage division 20 (10) = 4 volt 20 + 30 V2 = 15 (10) = 6 volt 15 + 10 V1 =

and,

Applying KVL V1 V2 + Vab = 0 4 6 + Vab = 0 VTh = Vab = 2 volt Thevenin Resistance :

Rab = [20 || 30 ] + [15 || 10 ] = 12 + 6 = 18 RTh = Rab = 18


SOL 5.1.26

Option (A) is a correct. Using source transformation of 24 V source

Adding parallel connected sources

Chap 5

Circuit Theorems

319

So,
SOL 5.1.27

VTh = 4 V , RTh = 6

Option (A) is correct. Thevenin voltage: (Open circuit voltage)

VTh = Thevenin resistance :

6 ( 40) 6+4 = 24 volt

(using voltage division)

SOL 5.1.28

RTh = 6 || 4 = 6 # 4 = 2.4 6+4 Option (B) is correct. For the circuit of figure (A)

320

Circuit Theorems

Chap 5

VTh = Va Vb Va = 24 V Vb = 6 ( 6) = 4 V 6+3 VTh = 24 ( 4) = 28 V For the circuit of figure (B), using source transformation

(Voltage division)

Combining parallel resistances, 12 || 4 = 3 Adding parallel current sources, 8 4 = 4 A (downward)

VTh = 12 V
SOL 5.1.29

Option (C) is correct. For the circuit for fig (A)

RTh = Rab = 6 || 3 = 2 For the circuit of fig (B), as obtained in previous solution.

Chap 5

Circuit Theorems

321

RTh = 3
SOL 5.1.30

Option (C) is correct.

Using current division

(5 + 1) (12) = 6 (12) 6+4 (5 + 1) + (3 + 1) = 7.2 A V1 = I1 # 1 = 7.2 V I1 = I2 =

(3 + 1) (12) = 4.8 A (3 + 1) + (5 + 1) V2 = 5I2 = 5 # 4.8 = 24 V VTh + V1 V2 = 0 VTh = V2 V1 = 24 7.2 = 16.8 V


SOL 5.1.31

(KVL)

Option (B) is correct. We obtain Thevenins resistance across a -b and then use source transformation of Thevenins circuit to obtain equivalent Norton circuit.

RTh = (5 + 1) || (3 + 1) = 6 || 4 = 2.4 Thevenins equivalent is

322

Circuit Theorems

Chap 5

Norton equivalent

SOL 5.1.32

Option (B) is correct.

The current source connected in parallel with load does not affect Thevenin equivalent circuit. Thus, Thevenin equivalent circuit will contain its usual form of a voltage source in series with a resistor.
SOL 5.1.33

Option (C) is correct. The network consists of resistor and dependent sources because if it has independent source then there will be an open circuit Thevenin voltage present. Option (D) is correct. Current I can be easily calculated by Thevenins equivalent across 6 . Thevenin voltage : (Open circuit voltage)

SOL 5.1.34

In the bottom mesh I2 = 1 A In the bottom left mesh

Chap 5

Circuit Theorems

323

VTh 12I2 + 3 = 0 VTh = 3 (12) (1) = 9 V Thevenin Resistance :

RTh = 12 so, circuit becomes as

(both 4 resistors are short circuit)

VTh = 9 = 9 = 0.5 A 18 RTh + 6 12 + 6 Note: The problem can be solved easily by a single node equation. Take the nodes connecting the top 4 , 3 V and 4 as supernode and apply KCL. I =
SOL 5.1.35

Option (D) is correct. Thevenin voltage (Open circuit voltage) :

Applying KCL at top middle node VTh 2Vx + VTh + 1 = 0 3 6 VTh 2VTh + VTh + 1 = 0 3 6 2VTh + VTh + 6 = 0 VTh = 6 volt

(VTh = Vx )

324

Circuit Theorems

Chap 5

Thevenin Resistance : Open circuit voltage VTh = Isc Short circuit current To obtain Thevenin resistance, first we find short circuit current through a -b RTh =

Writing KCL at top middle node Vx 2Vx + Vx + 1 + Vx 0 = 0 3 6 3 2Vx + Vx + 6 + 2Vx = 0 Vx = 6 volt Isc = Vx 0 = 6 = 2 A 3 3 Thevenins resistance, RTh = VTh = 6 = 3 2 Isc

Direct Method :
Since dependent source is present in the circuit, we put a test source across a -b to obtain Thevenins equivalent.

By applying KCL at top middle node Vx 2Vx + Vx + 1 + Vx Vtest = 0 3 6 3 2Vx + Vx + 6 + 2Vx 2Vtest = 0 2Vtest Vx = 6 Itest = Vtest Vx 3 3Itest = Vtest Vx Vx = Vtest 3Itest Put Vx into equation (i) We have

...(i)

Chap 5

Circuit Theorems

325

2Vtest (Vtest 3Itest) = 6 2Vtest Vtest + 3Itest = 6 Vtest = 6 3Itest For Thevenins equivalent circuit

...(ii)

Vtest VTh RTh Vtest Comparing equation VTh


SOL 5.1.36

= Itest = VTh + RTh Itest (ii) and (iii) = 6 V , RTh = 3 ...(iii)

Option (C) is correct. We obtain Thevenins equivalent across R. Thevenin voltage : (Open circuit voltage)

Applying KVL 18 6Ix 2Ix (1) Ix = 0 Ix = 18 = 2 A 9 VTh = (1) Ix = (1) (2) = 2 V Thevenin Resistance : RTh = VTh Isc

Isc " Short circuit current

326

Circuit Theorems

Chap 5

Ix = 0 So dependent source also becomes zero.

(Due to short circuit)

Isc = 18 = 3 A 6 Thevenin resistance, RTh = VTh = 2 3 Isc Now, the circuit becomes as

I =

2 =3 2+R 3 2 = 2 + 3R

R =0
SOL 5.1.37

Option (D) is correct.

Using voltage division V = VTh b From the table, 3 3 + RTh l 8 = VTh b 8 l 8 + RTh 6 = VTh b ...(i) ...(ii) R R + RTh l

Chap 5

Circuit Theorems

327

Dividing equation (i) and (ii), we get 6 = 3 (8 + RTh) 8 8 (3 + RTh) 6 + 2RTh = 8 + RTh RTh = 2 Substituting RTh into equation (i) 6 = VTh b 3 l 3+2 VTh = 10 V
SOL 5.1.38

Option (C) is correct. Norton current : (Short circuit current) The Norton equivalent current is equal to the short-circuit current that would flow when the load replaced by a short circuit as shown below

Applying KCL at node a I N + I1 + 2 = 0 I1 = 0 20 = 5 A a 6 24 So, IN 5 + 2 = 0 6 IN = 7 A 6 Norton resistance : Set all independent sources to zero (i.e. open circuit current sources and short circuit voltage sources) to obtain Nortons equivalent resistance RN .

RN = 24
SOL 5.1.39

Option (C) is correct. Using source transformation of 1 A source

328

Circuit Theorems

Chap 5

Again, source transformation of 2 V source

Adding parallel current sources

Alternate Method: Try to solve the problem using superposition method.


SOL 5.1.40

Option (C) is correct. Short circuit current across terminal a -b is

For simplicity circuit can be redrawn as

IN =

3 (10) 3+6 = 3.33 A

(Current division)

Chap 5

Circuit Theorems

329

Nortons equivalent resistance

RN = 6 + 3 = 9
SOL 5.1.41

Option (C) is correct.

The voltage across load terminal is simply Vs and it is independent of any other current or voltage. So, Thevenin equivalent is VTh = Vs and RTh = 0 (Voltage source is ideal). The Norton equivalent does not exist because of parallel connected voltage source.
SOL 5.1.42

Option (B) is correct.

The output current from the network is equal to the series connected current source only, so IN = Is . Thus, effect of all other component in the network does not change IN . In this case Thevenins equivalent is not feasible because of the series connected current source.
SOL 5.1.43

Option (C) is correct. Norton current : (Short circuit current)

330

Circuit Theorems

Chap 5

Using source transformation

Nodal equation at top center node 0 24 + 0 ( 6) + I = 0 N 6 3+3 4 + 1 + IN = 0 IN = 3 A Norton Resistance :

RN = Rab = 6 || (3 + 3) = 6 || 6 = 3 So, Norton equivalent will be

SOL 5.1.44

Option (C) is correct. We obtain Thevenins equivalent across R. By source transformation of both voltage sources

Adding parallel sources and combining parallel resistances

Chap 5

Circuit Theorems

331

Here, VTh = 5.4 V , RTh = 60 For maximum power transfer R = RTh = 60

Maximum Power absorbed by R ^VTh h2 (5.4) 2 P = = = 121.5 mW 4 # 60 4R Alternate Method: Thevenin voltage (open circuit voltage) may be obtained using node voltage method also.
SOL 5.1.45

Option (B) is correct.

V = Vs b Power absorbed by RL

RL Rs + R L l

2 (V) 2 = V s RL 2 RL (Rs + RL) From above expression, it is known that power is maximum when Rs = 0

PL =

332

Circuit Theorems

Chap 5

Note : Do not get confused with maximum power transfer theorem. According to maximum power transfer theorem if RL is variable and Rs is fixed then power dissipated by RL is maximum when RL = Rs .
SOL 5.1.46

Option (C) is correct. We solve this problem using maximum power transfer theorem. First, obtain Thevenin equivalent across RL . Thevenin Voltage : (Open circuit voltage)

Using source transformation

Using nodal analysis VTh 24 + VTh 24 = 0 6 2+4 2VTh 48 = 0 & VTh = 24 V Thevenin resistance :

RTh = 6 || 6 = 3 Circuit becomes as

Chap 5

Circuit Theorems

333

SOL 5.1.47

For maximum power transfer RL = RTh = 3 Value of maximum power (24) 2 (V ) 2 Pmax = Th = = 48 W 4#3 4RL Option (D) is correct. This can be solved by reciprocity theorem. But we have to take care that the polarity of voltage source have the same correspondence with branch current in each of the circuit. In figure (B) and figure (C), polarity of voltage source is reversed with respect to direction of branch current so V1 =V2 =V3 I1 I2 I3 I2 = I 3 = 2 A Option (C) is correct. According to reciprocity theorem in any linear bilateral network when a single voltage source Va in branch a produces a current Ib in branches b, then if the voltage source Va is removed(i.e. branch a is short circuited) and inserted in branch b, then it will produce a current Ib in branch a. So, I2 = I1 = 20 mA Option (A) is correct. According to reciprocity theorem in any linear bilateral network when a single current source Ia in branch a produces a voltage Vb in branches b, then if the current source Ia is removed(i.e. branch a is open circuited) and inserted in branch b, then it will produce a voltage Vb in branch a.

SOL 5.1.48

SOL 5.1.49

So,
SOL 5.1.50

V2 = 2 volt

Option (A) is correct. We use Millmans theorem to obtain equivalent resistance and voltage across a -b.

334

Circuit Theorems

Chap 5

96 + 40 + 80 Vab = 240 200 800 = 144 = 28.8 V 1 + 1 + 1 5 240 200 800 The equivalent resistance 1 = 96 1 + 1 + 1 240 200 800 Now, the circuit is reduced as Rab =

I =
SOL 5.1.51

28.8 = 100 mA 96 + 192

Option (C) is correct. First we obtain equivalent voltage and resistance across terminal a -b using Millmans theorem.

60 + b 120 l + 20 15 15 5 Vab = = 24 V 1 + 1 +1 15 15 5 1 = 3 1 + 1 +1 15 15 5 So, the circuit is reduced as Rab =

I = 24 = 3 A 3+5 ***********

Chap 5

Circuit Theorems

335

SOLUTIONS 5.2

SOL 5.2.1

Option (B) is correct. Thevenin Voltage: (Open circuit voltage): The open circuit voltage will be equal to V , i.e. VTh = V Thevenin Resistance: Set all independent sources to zero i.e. open circuit the current source and short circuit the voltage source as shown in figure

Open circuit voltage = V1


SOL 5.2.2

Option (C) is correct. Set all independent sources to zero as shown,

RTh = 6
SOL 5.2.3

Option (B) is correct. V is obtained using super position. Due to source V1 only : (Open circuit source I 3 and short circuit source V2 )

336

Circuit Theorems

Chap 5

50 (V ) = 1 V (using voltage division) 100 + 50 1 3 1 so, A=1 3 Due to source V2 only : (Open circuit source I 3 and short circuit source V1 ) V =

50 (V ) = 1 V 100 + 50 2 3 2 So, B =1 3 Due to source I 3 only : (short circuit sources V1 and V2 ) V =

(using voltage division)

V = I 3 [100 || 100 || 100] = I 3 b 100 l 3 So, C = 100 3 Alternate Method: Try to solve by nodal method, taking a supernode corresponding to voltage source V2 .
SOL 5.2.4

Option (D) is correct. We solve this problem using linearity and taking assumption that I = 1 A .

In the circuit,

V2 = 4I = 4 V I 2 = I + I1 = 1 + V2 = 1 + 4 = 4 A 12 3 4+8

(Using Ohms law) (Using KCL)

Chap 5

Circuit Theorems

337

SOL 5.2.5

V3 = 3I2 + V2 = 3# 4 +4 = 8V 3 Is = I 3 + I 2 = V3 + I2 = 8 + 4 = 4 A 3 3 3 Applying superposition When Is = 4 A , I = 1 A But actually Is = 2 A , So I = 1 # 2 = 0.5 A 4 Option (A) is correct. Solving with superposition, Due to 6 V source only : (Open circuit 2 mA source)

(Using KVL)

(Using KCL)

6 = 6 = 0.6 mA 6 + 6 || 12 6 + 4 I1 = 6 (Is) = 6 # 0.6 = 0.2 mA 6 + 12 18 Due to 2 mA source only : (Short circuit 6 V source) : Is =

(Using current division)

Combining resistances, 6 k || 6 k = 3 k 3 k + 6 k = 9 k

I2 =

9 ( 2) = 1.2 mA 9+6 I = I1 + I 2

(Current division) (Using superposition)

338

Circuit Theorems

Chap 5

= 0.2 1.2 = 1 mA Alternate Method: Try to solve the problem using source conversion.
SOL 5.2.6

Option (D) is correct. We find Thevenin equivalent across a -b.

SOL 5.2.7

VTh RTh + RL From the data given in table 10 = VTh RTh + 2 6 = VTh RTh + 10 Dividing equation (i) and (ii), we get 10 = RTh + 10 6 RTh + 2 10RTh + 20 = 6RTh + 60 4RTh = 40 & RTh = 10 Substituting RTh into equation (i) 10 = VTh 10 + 2 VTh = 10 (12) = 120 V For RL = 20 , = 120 = 4 A IL = VTh RTh + RL 10 + 20 Option (A) is correct. IL =

...(i) ...(ii)

For maximum power transfer RTh = RL = 2 To obtain RTh set all independent sources to zero and put a test source across the load terminals.

Chap 5

Circuit Theorems

339

RTh = Vtest Itest Using KVL, Vtest 4Itest 2Itest kVx 4Itest = 0 Vtest 10Itest k ( 2Itest) = 0 Vtest = (10 2k) Itest RTh = Vtest = 10 2k = 2 Itest 8 = 2k k =4
SOL 5.2.8

(Vx = 2Itest )

Option (D) is correct. To calculate maximum power transfer, first we will find Thevenin equivalent across load terminals. Thevenin voltage: (Open circuit voltage)

using source transformation

340

Circuit Theorems

Chap 5

VTh = Thevenin resistance :

2 (24) 2+2 = 12 V

(using voltage division)

RTh = 1 + 2 || 2 = 1 + 1 = 2 k circuit becomes as

RL V RTh + RL Th For maximum power transfer RL = RTh VL = VTh # RTh = VTh 2 2RTh So maximum power absorbed by RL VL =
2 2 (12) 2 Pmax = V L = VTh = = 18 mW 4#2 RL 4RTh Option (C) is correct. The circuit with Norton equivalent

SOL 5.2.9

IN + I = V RN I = V IN RN From the given graph, the equation of line I = 2V 6 Comparing with general form So,

(General form)

Chap 5

Circuit Theorems

341

1 = 2 or R = 0.5 N RN IN = 6 A
SOL 5.2.10

Option (D) is correct. Thevenin voltage: (Open circuit voltage)

VTh = 4 + ^2 # 2h = 4 + 4 = 8 V Thevenin Resistance:

RTh = 2 + 3 = 5 = RN Norton current: IN = VTh = 8 A 5 RTh Option (A) is correct. If we solve this circuit directly by nodal analysis, then we have to deal with three variables. We can replace the left most and write most circuit by their Thevenin equivalent as shown below.

SOL 5.2.11

342

Circuit Theorems

Chap 5

Now the circuit becomes as shown

Writing node equation at the top center node V1 4 + V1 + V1 12 = 0 1+1 6 1+2 V1 + 4 + V1 + V1 12 = 0 2 6 3 3V1 12 + V1 + 2V1 24 = 0 6V1 = 36 V1 = 6 V
SOL 5.2.12

Option (A) is correct. The circuit is as shown below

When RL = 50 , power absorbed in load will be 2 R b R +s 50 Is l 50 = 20 kW s

...(i)

SOL 5.2.13

When RL = 200 , power absorbed in load will be 2 Rs ...(ii) b Rs + 200 Is l 200 = 20 kW Dividing equation (i) and (ii), we have (Rs + 200) 2 = 4 (Rs + 50) 2 Rs = 100 and Is = 30 A From maximum power transfer, the power supplied by source current Is will be maximum when load resistance is equal to source resistance i.e. RL = Rs . Maximum power is given as 2 (30) 2 # 100 = 22.5 kW Pmax = I s Rs = 4 4 Option (C) is correct. Norton current, IN = 0 because there is no independent source present in the circuit. To obtain Norton resistance we put a 1 A test source across the load terminal as shown in figure.

Chap 5

Circuit Theorems

343

SOL 5.2.14

Norton or Thevenin resistance RN = Vtest 1 Writing KVL in the left mesh 20I1 + 10 ^1 I1h 30I1 = 0 20I1 10I1 30I1 + 10 = 0 I1 = 0.5 A Writing KVL in the right mesh Vtest 5 ^1 h 30I1 = 0 Vtest 5 30 ^0.5h = 0 Vtest 5 15 = 0 RN = Vtest = 20 1 Option (C) is correct. In circuit (b) transforming the 3 A source in to 18 V source all source are 1.5 times of that in circuit (a) as shown in figure.

Using principal of linearity, Ib = 1.5Ia


SOL 5.2.15

Option (B) is correct. 6 and 3 resistors are in parallel, which is equivalent to 2 .

344

Circuit Theorems

Chap 5

Using source transformation of 6 A source

Source transform of 4 A source

Adding series resistors and sources on the left

Source transformation of 48 V source

Source transformation of 4 A source. 3

Chap 5

Circuit Theorems

345

I = 12 + 72 + Vs 19 + 9 Vs = (28 # I) 12 72 = (28 # 5) 12 72 = 56 V
SOL 5.2.16

Option (A) is correct. We obtain I using superposition. Due to 24 V source only : (Open circuit 6 A)

Applying KVL 24 6I1 3I1 3I1 = 0 I1 = 24 = 2 A 12 Due to 6 A source only : (Short circuit 24 V source)

Applying KVL to supermesh 6I2 3 (6 + I2) 3I2 = 0 6I2 + 18 + 3I2 + 3I2 = 0 I2 = 18 = 3 A 12 2 I = I1 + I 2 = 23 = 1 A 2 2

From superposition,

346

Circuit Theorems

Chap 5

Alternate Method: Note that current in 3 resistor is ^I + 6h A, so by applying KVL around the outer loop, we can find current I .
SOL 5.2.17

Option (B) is correct.

I = From the table, 2=

VTh R + RTh ...(i) ...(ii)

VTh 3 + RTh 1.6 = VTh 5 + RTh Dividing equation (i) and (ii), we get 2 = 5 + RTh 1.6 3 + RTh 6 + 2RTh = 8 + 1.6RTh 0.4RTh = 2 RTh = 5 Substituting RTh into equation (i) 2 = VTh 3+5 VTh = 2 (8) = 16 V
SOL 5.2.18

Option (D) is correct. We have, VTh RTh + R VTh = 16 V , RTh = 5 I = 16 = 1 5+R 16 = 5 + R R = 11 I =

SOL 5.2.19

Option (B) is correct.

Chap 5

Circuit Theorems

347

It can be solved by reciprocity theorem. Polarity of voltage source should have same correspondence with branch current in each of the circuit. Polarity of voltage source and current direction are shown below V1 =V2 = V3 So, I1 I2 I3 10 = 20 = 40 2.5 I2 I3 I2 = 5 A I 3 = 10 A
SOL 5.2.20

Option (B) is correct. Open circuit voltage RTh = Voc = Isc short circuit Thevenin voltage: (Open circuit voltage Voc ) Using source transformation of the dependent source

Applying KCL at top left node 24 = Vx & Vx = 144 V 6 Using KVL, Vx 8I Vx Voc = 0 2 144 0 144 = Voc 2 Voc = 72 V Short circuit current (Isc ):

Applying KVL in the right mesh Vx 8Isc Vx = 0 2

348

Circuit Theorems

Chap 5

Vx = 8I sc 2 Vx = 16Isc KCL at the top left node V Vx /2 24 = Vx + x 6 8 24 = Vx + Vx 6 16 Vx = 1152 V 11 Isc = Vx = 1152 = 72 A 16 11 # 16 11 RTh = Voc = 72 = 11 72 Isc b 11 l Alternate method : We can obtain Thevenin equivalent resistance without calculating the Thevenin voltage (open circuit voltage). Set all independent sources to zero (i.e. open circuit current sources and short circuit voltage sources) and put a test source Vtest between terminal a -b as shown

RTh = Vtest Itest 6I + 8I Vx Vtest = 0 2 6I V = 0 14I test 2 11I = Vtest So


SOL 5.2.21

(KVL) Vx = 6Itest (Using Ohms law)

RTh = Vtest = 11 Itest Option (C) is correct. We solve this problem using linearity and assumption that I = 1 A .

Chap 5

Circuit Theorems

349

V1 = 4I + 2I =6V I 2 = I1 + I = V1 + I = 6 + 1 = 2.5 A 4 4 V2 = 4I2 + V1 = 4 (2.5) + 6 = 16 V Is + I 3 = I 2 Is

(Using KVL) (Using KCL)

(Using KVL) (Using KCL)

SOL 5.2.22

V2 = I 2 4 + 12 Is = 16 + 2.5 = 3.5 A 16 When Is = 3.5 A , I = 1A But Is = 14 A , so I = .1 # 14 = 4 A 3.5 Option (A) is correct. To obtain V -I equation we find the Thevenin equivalent across the terminal at which X is connected. Thevenin voltage : (Open circuit voltage)

V1 = 6 # 1 = 6 V 12 + V1 V3 = 0 V3 = 12 + 6 = 18 V VTh V2 V3 = 0 VTh = V2 + V3 VTh = 2 + 18 = 20 V Thevenin Resistance :

(KVL in outer mesh) (KVL in Bottom right mesh) (V2 = 2 # 1 = 2 V)

350

Circuit Theorems

Chap 5

RTh = 1 + 1 = 2 Now, the circuit becomes as

I = V VTh RTh V = RTh I + VTh so A = RTh = 2 B = VTh = 20 V Alternate Method:

In the mesh ABCDEA, we have KVL equation as V 1 (I + 2) 1 (I + 6) 12 = 0 V = 2I + 20 So, A = 2, B = 2


SOL 5.2.23

Option (C) is correct. This problem will easy to solve if we obtain Thevenin equivalent across the 12 V source. Thevenin voltage : (Open circuit voltage)

Chap 5

Circuit Theorems

351

Mesh currents are Mesh 1: I1 = 0 Mesh 2: I1 I 3 = 2 or I 3 = 2 A Mesh 3: I 3 I2 = 4 or I2 = 6 A Mesh equation for outer loop VTh 1 # I 3 1 # I2 = 0 VTh ( 2) ( 6) = 0 VTh + 2 + 6 = 0 VTh = 8 V Thevenin resistance :

(due to open circuit)

RTh = 1 + 1 = 2 circuit becomes as

12 ( 8) I = 12 VTh = = 10 A 2 RTh Power supplied by 12 V source P12 V = 10 # 12 = 120 W Alternate Method:

352

Circuit Theorems

Chap 5

KVL in the loop ABCDA 12 1 (I 2) 1 (I 6) = 0 2I = 20 I = 10 A Power supplied by 12 V source P12 V = 10 # 12 = 120 W


SOL 5.2.24

Option (A) is correct. To obtain V -I relation, we obtain either Norton equivalent or Thevenin equivalent across terminal a -b. Norton Current (short circuit current) :

Applying nodal analysis at center node IN + 2 = 24 4 IN = 6 2 = 4 A Norton Resistance :

RN = 4 Now, the circuit becomes as

(Both 2 resistor are short circuited)

IN = V + I RN

Chap 5

Circuit Theorems

353

or

4 =V +I 4 16 = V + 4I V = 4I + 16

Alternate Method: Solve by writing nodal equation at the center node.


SOL 5.2.25

Option (A) is correct. For maximum power transfer RL = RTh . To obtain Thevenin resistance set all independent sources to zero and put a test source across load terminals.

RTh = Vtest Itest Writing KCL at the top center node Vtest + Vtest 2Vx = I test 2k 1k Also, Vtest + Vx = 0 so Vx = Vtest Substituting Vx = Vtest into equation (i) Vtest + Vtest 2 ( Vtest) = I test 2k 1k Vtest + 6Vtest = 2Itest RTh = Vtest = 2 k - 286 7 Itest
SOL 5.2.26

...(i) (KVL in left mesh)

Option (A) is correct. Redrawing the circuit in Thevenin equivalent form

354

Circuit Theorems

Chap 5

I = VTh V RTh or, V = RTh I + VTh From the given graph V = 4I + 8 So, by comparing RTh = 4 k , VTh = 8 V For maximum power transfer RL = RTh Maximum power absorbed by RL
2

(General form)

SOL 5.2.27

2 (8) Pmax = VTh = = 4 mW 4#4 4RTh Option (C) is correct. To fine out Thevenin equivalent of the circuit put a test source between node a and b,

RTh = Vtest Itest Writing node equation at V1 V1 Ix + V1 = I x 1 1 2V1 = ^1 + h Ix Ix is the branch current in 1 resistor given as Ix = Vtest V1 1 V1 = Vtest Ix Substituting V1 into equation (i) 2 ^Vtest Ix h = ^1 + h Ix 2Vtest = ^3 + h Ix 2Vtest = ^3 + h Itest RTh = Vtest = 3 + = 3 2 Itest 3+ = 6 = 3
SOL 5.2.28

...(i)

^Ix = Itest h

Option (C) is correct. Let Thevenin equivalent of both networks are as shown below.

Chap 5

Circuit Theorems

355

P = b VTh l R RTh + R 2 VTh Pl = R f R + RTh p 2 2 VTh = 4b lR 2R + RTh Thus P < Pl < 4P


2

(Single network N ) (Two N are added)

SOL 5.2.29

Option (C) is correct. I1 = P1 and I = 2 R P2 R

Using superposition I = I1 ! I 2 P1 ! P2 R R 2 I R = ( P1 ! P2 ) 2 =
SOL 5.2.30

Option (B) is correct. From the substitution theorem we know that any branch within a circuit can be replaced by an equivalent branch provided that replacement branch has the same current through it and voltage across it as the original branch. The voltage across the branch in the original circuit

40 || 60 (using voltage division) (20) (40 || 60) + 16 = 24 # 20 = 12 V 40 Current entering terminal a -b is I = V = 12 = 200 mA R 60 In fig(B), to maintain same voltage V = 12 V current through 240 resistor must V =

356

Circuit Theorems

Chap 5

be IR = 12 = 50 mA 240 By using KCL at terminal a , as shown

I = I R + IS 200 = 50 + Is Is = 150 mA ,
SOL 5.2.31

down wards

Option (B) is correct. Thevenin voltage : (Open circuit voltage) In the given problem, we use mesh analysis method to obtain Thevenin voltage

I3 = 0 Writing mesh equations Mesh 1: 36 12 (I1 I2) 6 (I1 I 3) 36 12I1 + 12I2 6I1 3I1 2I2 Mesh 2: 24I2 20 (I2 I 3) 12 (I2 I1) 24I2 20I2 12I2 + 12I1 14I2 From equation (i) and (ii) I1 = 7 A, I2 = 1 A 3 2 Mesh 3: 6 (I 3 I1) 20 (I 3 I2) VTh

(a -b is open circuit)

=0 =0 =6 =0 =0 = 3I1

(I 3 = 0) ...(i)

(I 3 = 0) ...(ii)

=0

Chap 5

Circuit Theorems

357

6 :0 7 D 20 :0 1 D VTh = 0 3 2 14 + 10 = VTh VTh = 24 volt Thevenin Resistance :

RTh = (20 + 4) || 24 = 24 || 24 = 12 Alternate Method: VTh can be obtained by writing nodal equation at node a and at center node.
SOL 5.2.32

Option (C) is correct. We obtain Thevenins equivalent across load terminal. Thevenin voltage : (Open circuit voltage)

Using KCL at top left node 5 = Ix + 0

358

Circuit Theorems

Chap 5

Ix = 5 A Using KVL 2Ix 4Ix VTh = 0 2 (5) 4 (5) = VTh VTh = 10 volt Thevenin Resistance : First we find short circuit current through a -b

Using KCL at top left node 5 = Ix + Isc Ix = 5 Isc Applying KVL in the right mesh 2Ix 4Ix + 0 = 0 Ix = 0 So, 5 Isc = 0 or Isc = 5 A Thevenin resistance, RTh = VTh = 10 = 2 5 Isc Now, the circuit becomes as

V = VTh b So,
SOL 5.2.33

R R + RL l V = VTh = 10 volt R = RTh = 2

(Using voltage division)

Option (C) is correct. We obtain Thevenin equivalent across the load terminals Thevenin Voltage : (Open circuit voltage)

Chap 5

Circuit Theorems

359

VTh = Va Vb Rotating the circuit, makes it simple

340 (40) 340 + 60 = 34 A Va = 20I1 = 20 # 34 = 680 V I1 = Similarly, 60 (40) = 6 A 60 + 340 Vb = 100I2 = 100 # 6 = 600 V Thevenin voltage VTh = 680 600 = 80 V Thevenin Resistance : I2 =

(Current division) (Ohms Law) (Current division) (Ohms Law)

RTh = 16 + (240 + 40) || (20 + 100) = 16 + (280 || 120) = 16 + 84 = 100

360

Circuit Theorems

Chap 5

Now, circuit reduced as

For maximum power transfer RL = RTh = 100 Maximum power transferred to RL (V ) 2 (80) 2 Pmax = Th = 4 # 100 4RL = 16 W
SOL 5.2.34

Option (A) is correct. We use source transformation as follows

Chap 5

Circuit Theorems

361

I = 36 12 = 3 A 6+2 Power supplied by 36 V source P36 V = 3 # 36 = 108 W


SOL 5.2.35

Option (D) is correct. Now, we do source transformation from left to right as shown

Vs = (27 + 1.5) (4 || 2 ) = 28.5 # 4 3 = 38 V Power supplied by 27 A source P27 A = Vs # 27 = 38 # 27 = 1026 W


SOL 5.2.36

Option (C) is correct. First, we find current I in the 4 resistors using superposition.

362

Circuit Theorems

Chap 5

Due to 18 V source only : (Open circuit 4 A and short circuit 12 V source)

I1 = 18 = 4.5 A 4 Due to 12 V source only : (Open circuit 4 A and short circuit 18 V source)

I2 = 12 = 3 A 4 Due to 4 A source only : (Short circuit 12 V and 18 V sources)

I3 = 0 So, I = I1 + I2 + I 3 = 4.5 3 + 0 = 1.5 A Power dissipated in 4 resistor P4 = I 2 (4) = (1.5) 2 # 4 = 9 W

(Due to short circuit)

Chap 5

Circuit Theorems

363

Alternate Method: Let current in 4 resistor is I , then by applying KVL around the outer loop 18 12 4I = 0 I = 6 = 1.5 A 4 So, power dissipated in 4 resistor P4 = I 2 (4) = (1.5) 2 # 4 = 9 W
SOL 5.2.37

Option (D) is correct. We obtain Thevenin equivalent across terminal a -b. Thevenin Voltage : Since there is no independent source present in the network, Thevenin voltage is simply zero. VTh = 0 Thevenin Resistance : Put a test source across terminal a -b

RTh = Vtest Itest For the super node V1 Vtest = 2000Ix V1 Vtest = 2000 b V1 l 4000 V1 = V or V = 2V test 1 test 2 Applying KCL to the super node V1 0 + V1 + Vtest = I test 4k 4k 4k 2V1 + Vtest = 4 # 103 Itest 2 (2Vtest) + Vtest = 4 # 103 Itest Vtest = 4 # 103 = 800 5 Itest
SOL 5.2.38

^Ix = V1 /4000h

(V1 = 2Vtest)

Option (C) is correct. Using, Thevenin equivalent circuit Thevenin Voltage : (Open circuit voltage)

364

Circuit Theorems

Chap 5

Ix = 4 A Writing KVL in bottom right mesh 24 (1) Ix VTh = 0 VTh = 24 + 4 = 20 V Thevenin resistance : open circuit voltage Voc RTh = = short circuit current Isc Voc = VTh = 20 V Isc is obtained as follows

(due to open circuit)

= 24 = 24 A 1 = Isc = Isc = 20 A RTh = 20 = 1 20 The circuit is as shown below Ix Ix + 4 24 + 4 Isc

(using KCL)

1 (V ) = 1 ( 20)= 10 volt (Using voltage division) 1+1 1 + RTh Th Alternate Method: Note that current in bottom right most 1 resistor is ^Ix + 4h, V =

Chap 5

Circuit Theorems

365

so applying KVL around the bottom right mesh, 24 Ix ^Ix + 4h = 0 Ix = 14 A So, V = 1 # ^Ix + 4h = 14 + 4 = 10 V
SOL 5.2.39

Option (A) is correct. Writing currents into 100 and 300 resistors by using KCL as shown in figure.

SOL 5.2.40

Ix = 1 A, Vx = Vtest Writing mesh equation for bottom right mesh. Vtest = 100 (1 2Ix ) + 300 (1 2Ix 0.01Vx ) + 800 = 100 V RTh = Vtest = 100 1 Option (D) is correct. For RL = 10 k , Vab1 = 10k # 3.6m = 6 V For RL = 30 k , Vab2 = 30k # 4.8m = 12 V 10 V = 6 ...(i) Vab1 = 10 + RTh Th 30 V = 12 ...(ii) Vab2 = 30 + RTh Th Dividing equation (i) and (ii), we get RTh = 30 k . Maximum power will be transferred when RL = RTh = 30 k . Option (C) is correct. Equation for V -I can be obtained with Thevenin equivalent across a -b terminals. Thevenin Voltage: (Open circuit voltage)

SOL 5.2.41

366

Circuit Theorems

Chap 5

Writing KCL at the top node Vx = VTh Vx 20 40 Vx = 2VTh 2Vx 3Vx = 2VTh & Vx = 2 VTh 3 KCL at the center node Vx VTh + Vx = 0.3 20 30 3Vx 3VTh + 2Vx = 18 5Vx 3VTh = 18 2 V 3V = 18 2 5 b l Th Th bVx = 3 VTh l 3 10VTh 9VTh = 54 VTh = 54 volt Thevenin resistance : When a dependent source is present in the circuit the best way to obtain Thevenin resistance is to remove all independent sources and put a test source across a -b terminals as shown in figure.

RTh = Vtest Itest KCL at the top node Vx + I = Vtest test 20 + 30 40 Vx + I = Vtest test 50 40 Vx = 30 (Vtest) 30 + 20 = 3 Vtest 5 Substituting Vx into equation (i), we get 3Vtest + I = Vtest test 50 5 (40) Itest = Vtest b 1 3 l = Vtest 50 200 200 RTh = Vtest = 200 Itest

...(i) (using voltage division)

Chap 5

Circuit Theorems

367

The circuit now reduced as

I = V VTh = V 54 200 RTh V = 200I + 54


SOL 5.2.42

Option (D) is correct. To obtain Thevenin resistance put a test source across the terminal a , b as shown.

Vtest = Vx , Itest = Ix By writing loop equation for the circuit Vtest = 600 (I1 I2) + 300 (I1 I 3) + 900 (I1) Vtest = (600 + 300 + 900) I1 600I2 300I 3 ...(i) Vtest = 1800I1 600I2 300I 3 The loop current are given as, I1 = Itest , I2 = 0.3Vs , and I 3 = 3Itest + 0.2Vs Substituing theses values into equation (i), Vtest = 1800Itest 600 (0.01Vs) 300 (3Itest + 0.01Vs) Vtest = 1800Itest 6Vs 900Itest 3Vs 10Vtest = 900Itest , Vtest = 90Itest Thevenin resistance RTh = Vtest = 90 Itest Thevenin voltage or open circuit voltage will be zero because there is no independent source present in the network, i.e. Voc = 0 V ***********

368

Circuit Theorems

Chap 5

SOLUTIONS 5.3

SOL 5.3.1

Option (C) is correct. When 10 V is connected at port A the network is

Now, we obtain Thevenin equivalent for the circuit seen at load terminal, let Thevenin voltage is VTh, 10 V with 10 V applied at port A and Thevenin resistance is RTh .

IL = For RL = 1 , IL = 3 A 3=

VTh,10 V RTh + RL ...(i)

VTh,10 V RTh + 1 For RL = 2.5 , IL = 2 A V 2 = Th,10 V RTh + 2.5 Dividing above two 3 = RTh + 2.5 2 RTh + 1 3RTh + 3 = 2RTh + 5 RTh = 2 Substituting RTh into equation (i)

...(ii)

Chap 5

Circuit Theorems

369

VTh,10 V = 3 (2 + 1) = 9 V Note that it is a non reciprocal two port network. Thevenin voltage seen at port B depends on the voltage connected at port A. Therefore we took subscript VTh,10 V . This is Thevenin voltage only when 10 V source is connected at input port A. If the voltage connected to port A is different, then Thevenin voltage will be different. However, Thevenins resistance remains same. Now, the circuit is

For RL = 7 ,
SOL 5.3.2

IL =

VTh,10 V = 9 = 1A 2 + RL 2 + 7

Option (B) is correct. Now, when 6 V connected at port A let Thevenin voltage seen at port B is VTh,6 V . Here RL = 1 and IL = 7 A 3

VTh, 6 V = RTh # 7 + 1 # 7 = 2 # 7 + 7 = 7 V 3 3 3 3 This is a linear network, so VTh at port B can be written as VTh = V1 + where V1 is the input applied at port A. We have V1 = 10 V , VTh,10 V = 9 V ...(i) 9 = 10 + ` When V1 = 6 V , VTh, 6 V = 9 V ...(ii) ` 7 = 6 + Solving (i) and (ii) = 0.5 , = 4 Thus, with any voltage V1 applied at port A, Thevenin voltage or open circuit voltage at port B will be So, VTh, V = 0.5V1 + 4 For V1 = 8 V (open circuit voltage) VTh,8 V = 0.5 # 8 + 4 = 8 = Voc
1

370
SOL 5.3.3

Circuit Theorems

Chap 5

Option (C) is correct. Power transferred to RL will be maximum when RL is equal to the Thevenin resistance seen at the load terminals. To obtain Thevenin resistance, we set all independent sources zero(i.e. short circuit voltage source and open circuit current source) as shown in figure.

RTh = ^10 || 10h + 10 = 10 # 10 + 10 = 15 10 + 10


SOL 5.3.4

Option (C) is correct. For maximum power transfer, the load resistance RL must be equal to Thevenin resistance RTh seen at the load terminals. i.e. RL = RTh . Thevenin resistance is given by Open circuit voltage Voc RTh = = Short circuit current Isc The open circuit voltage can be obtained using the circuit shown below

The open circuit voltage is Voc = 100 V. Short circuit current is determined using following circuit

Chap 5

Circuit Theorems

371

From figure,

I1 = 100 = 12.5 A 8 Vx = 4 # 12.5 = 50 V

I2 = 100 + Vx = 100 50 = 12.5 A 4 4 Isc = I1 + I2 = 25 A So, RTh = Voc = 100 = 4 Isc 25 Thus, for maximum power transfer RL = RTh = 4 .
SOL 5.3.5

Option (D) is correct. Open circuit voltage (Voc) VTh = Isc Short circuit current (Isc) is voltage across node also. Applying nodal analysis we get RTh =

Here VTh

VTh + VTh + VTh 2i = 2 2 1 1 From the circuit, i = VTh = VTh 1 Therefore, VTh + VTh + VTh 2VTh = 2 2 1 1 or, VTh = 4 volt From the figure shown below it may be easily seen that the short circuit current at terminal XY is Isc = 2 A because i = 0 due to short circuit of 1 resistor and all current will pass through short circuit.

Therefore
SOL 5.3.6

Rth = VTh = 4 = 2 Isc 2

Option (C) is correct. Maximum power will be transferred when RL = RTh = 100 In this case voltage across RL is 5 V, therefore

372

Circuit Theorems
2 (10) 2 Pmax = VTh = = 0.25 W 4R 4 # 100 Option (B) is correct. Open circuit voltage VTh RTh = = Isc Short circuit current Thevenin voltage (Open circuit voltage):

Chap 5

SOL 5.3.7

Applying KCL at node we get VTh + VTh 10 = 1 5 5 or, VTh = 7.5 Short Circuit Current: Short circuit current through terminal a , b is obtained as follows.

Isc = 1 + 10 = 3 A 5

Thevenin resistance, RTh = VTh = 7.5 = 2.5 Isc 3 Note: Here current source being in series with dependent voltage source makes it ineffective.
SOL 5.3.8

Option (A) is correct. For maximum power delivered, load resistance RL must be equal to Thevenin resistance RTh seen from the load terminals. Open circuit voltage (Voc) VTh RTh = = Isc Short circuit current (Isc)

Chap 5

Circuit Theorems

373

Applying KCL at Node, we get 0.5I1 = VTh + I1 20 or VTh + 10I1 = 0 but I1 = VTh 50 40 Thus, VTh + VTh 50 = 0 4 or VTh = 10 V For Isc the circuit is shown in figure below.

Isc = 0.5I1 I1 = 0.5I1 but I1 = 50 = 1.25 A 40 Isc = 0.5 # 12.5 = 0.625 A So, Rth = VTh = 10 = 16 Isc 0.625 Alternate Method: Thevenin resistance can be obtained by setting all independent source to zero and put a test source across the load terminals as shown.

Writing KCL at top node Vtest + Vtest = I + 0.5I test 1 20 40 3 V = I + 0.5 Vtest b 40 l test 40 test 3 1 b 40 80 l Vtest = Itest 1 V =I test 16 test

^I1 = Vtest /40h

374

Circuit Theorems

Chap 5

Thevenin resistance, RTh = Vtest = 16 Itest Option (C) is correct. This can be solved by reciprocity theorem. But we have to take care that the polarity of voltage source have the same correspondence with branch current in each of the circuit. In figure (B) and figure (C), polarity of voltage source is reversed with respect to direction of branch current so V1 =V2 I1 I2 10 = 20 2 I I = 4 A Option (C) is correct. For maximum power transfer RL should be equal to RTh at same terminal. To obtain RTh set all independent sources to zero as shown below

SOL 5.3.9

SOL 5.3.10

RTh = ^5 || 20 h + 4 = 5 # 20 + 4 = 4 + 4 = 8 5 + 20 Option (A) is correct. Superposition theorem is applicable to only linear circuits. Option (D) is correct. V can not be determined without knowing the elements in box. Option ( ) is correct. Thevenin Voltage (open circuit voltage) :

SOL 5.3.11

SOL 5.3.12

SOL 5.3.13

Chap 5

Circuit Theorems

375

Writing KCL VTh 10 = 4V s 2 VTh = 8Vs + 10 10 VTh = Vs From equation (i) and (ii) VTh = 8 ^10 VTh h + 10 = 80 8VTh + 10 = 10 V Thevenin resistance : RTh = VTh Isc Isc is short circuit current through terminal A, B

...(i) ...(ii)

Isc = 10 Vs 4 Writing KCL at top center node Vs + 4V = I s sc 2 9V = I sc 2 s Vs = 2 Isc 9 Substituting Vs into equation (iii) 4Isc = 10 2 Isc 9 Substituting Vs in to equation (i) 4Isc = 10 2 Isc 9 38 I = 10 9 sc Isc = 90 A 38 RTh = 10 = 38 A 9 90/38 None of the option is correct.
SOL 5.3.14

...(iii)

Option (B) is correct. Using source transformation

376

Circuit Theorems

Chap 5

So,
SOL 5.3.15

IN = 2 A RN = 4.5

Option (B) is correct. Using source transformation

Adding parallel connected current source and combining the resistance I = 10 5 = 5 A 6 4 12 R = 12 = 2.4 5

SOL 5.3.16

Option (B) is correct. To obtain equivalent Thevenin resistance put a test source across A, B and set independent source to zero.

Chap 5

Circuit Theorems

377

RTh = Vtest Itest Simplifying above circuit we have

Writing node equation at top right node Vtest + 3VAB + Vtest = I test 1k 1k Vtest + 3Vtest + Vtest = I test 1000 1000 5Vtest = 1000Itest RTh = Vtest = 200 = 0.2 k Itest
SOL 5.3.17

^VAB = Vtest h

Option (D) is correct. Thevenin voltage or open circuit voltage across A, B can be computed using the circuit below.

Writing node equation at node x ^VTh + 3VAB h 5 VTh + 3VAB VTh + + 2k 2k 1k VTh + 3VAB 5 + VTh + 3VAB + 2VTh 10VTh 5 VTh
SOL 5.3.18

=0 =0 =0 = 0.5 V

^VAB = VTh h

Option (B) is correct. V + I = 100

...(i)

378

Circuit Theorems

Chap 5

Applying KVL in the loop V 1I = 0 From equation (i) and (ii) 2I = 100 & I = 50 A
SOL 5.3.19

...(ii)

Option (A) is correct.

Power transferred to the load 2 10 P = I 2 RL = b l RL RTh + RL For maximum power transfer RTh , should be minimum. RTh = 6R = 0 6+R R =0 Note : Do not get confused with maximum power transfer theorem. According to maximum power transfer theorem if RL is variable and RTh is fixed then power dissipated by RL is maximum when RL = RTh .
SOL 5.3.20

Option (A) is correct. Let Thevenin equivalent voltage of dc network is VTh and Thevenin resistance is RTh .

R V R + RTh Th 10 V 20 = 10 + RTh Th 20 V 30 = 20 + RTh Th Dividing equation (i) and (ii) 2 = 10 c 20 + RTh m 20 10 + RTh 3 40 + 4RTh = 60 + 3RTh VR =

...(i) ...(ii)

Chap 5

Circuit Theorems

379

SOL 5.3.21

RTh = 20 Substituting RTh into equation (i) 20 = 10 VTh 10 + 20 VTh = 60 V For R = 80 , VR = 80 ^60h = 48 V 80 + 20 Option (C) is correct. We obtain Thevenin equivalent across R. Thevenin voltage (Open circuit voltage) :

VTh = ^6 # 1h + 10 = 16 V Thevenin resistance :

SOL 5.3.22

RTh = 1 For maximum power transfer R = RTh = 1 The maximum power will be 2 ^16h2 Pmax = VTh = = 64 W 4 4RTh Option (B) is correct. Transforming the 5 A current source into equivalent voltage source

Writing node equation V1 10 + V1 = 2 12 5

380

Circuit Theorems

Chap 5

SOL 5.3.23

5V1 50 + 12V1 = 120 17V1 = 170 V1 = 10 V Current in 5 resistor I5 = V1 = 10 = 2 A 5 5 Option (C) is correct. Let the circuit is

Short circuit current Isc = 75 mA

Vt = 0.6 ,

Isc = VTh = 75 mA RTh I = 70 mA

...(i)

I = VTh 0.6 = 70 mA RTh VTh 0.6 = 70 # 103 RTh From equation (i) and (ii) 75 # 103 RTh 0.6 = 70 # 103 RTh 5 # 103 RTh = 0.6 RTh = 120

...(ii)

Chap 5
SOL 5.3.24

Circuit Theorems

381

Option (C) is correct.

SOL 5.3.25

I = 10 3 = 7 2 + RL 2 + RL Power delivered from source to load will be sum of power absorbed by RL and power absorbed by 3 V source 2 P = c 7 m RL + c 7 m # 3 2 + RL 2 + RL 49RL + 21 ^2 + RL h = ^2 + RL h2 ^42 + 70RL h = ^2 + RL h2 For maximum power transfer dP = 0 dRL 2 ^2 + RL h 60 + 70@ ^42 + 70RL h64 ^2 + RL h@ =0 ^2 + RL h4 ^2 + RL h^70h ^42 + 70RL h^2 h = 0 140 + 70RL 84 140RL = 0 RL = 4 = 0.8 5 Option (A) is correct. Transforming 10 V source into equivalent current source

Current in the circuit

10 || 10 = 5 10 A 1 A = 9 A

382

Circuit Theorems

Chap 5

I =

5 9 5 + 10 ^ h = 3A ***********

(Using current division)

Вам также может понравиться